Вы находитесь на странице: 1из 159

SERIES OF REAL NUMBERS

In this part of the course, we will be


concerned with how one can formalise the
idea of summing an infinite list of numbers

a1 + a2 + a3 + . . . .
As you would expect, this will once again
involve the notion of a limit.
We begin with a basic definition:

Definition Let (an)∞


n=1 be a sequence. For
each n ≥ 1, let
n
X
sn = a1 + a2 + . . . + a n = ak .
k=1
The series with nth term an, denoted an ,
P

is, formally, the ordered pair ((an), (sn)).

an is called the nth term of the series and sn


is called the nth partial sum of the series.
Using this ordered pair notation to denote a
series would be cumbersome and this is why
we denote a series by the notation an. But
P

remember, that although this looks like an


arithmetic expression, it is simply a notation
which we use to denote an ordered pair of
sequences.
Study of series is basically equivalent to the
study of sequences. If we have an infinite
series of real numbers

a1 + a2 + a3 + . . . ,

then we can study it via the sequence (sn),


where

sn = a1 + a2 + . . . + an .
Note that, as far as we are concerned, a
series involves an infinite list of numbers.
We do not discuss ‘finite’ series, since there
are no convergence issues there.
Example (−1)n.
P

The nth term of this sequence is (−1)n and


the nth partial sum sn is −1 if n is odd and
0 if n is even.
Example (−1)2n.
P

The nth term of this sequence is 1 and the


nth partial sum is n.
Note that the sequence (an) for the second
series is actually a subsequence of that for
the first - but that the partial sums bear
little resemblance.
n−1
1

X
Consider   .
2

2 n−1
1 1 1
 

sn = 1 + + + ··· + 
2 2 2
1 − (1/2)n
=
1− (1/2)
 n
1 
= 2 1−
  .
2
Convergence of series

Definition Let an be a series. If the


P

sequence (sn) of partial sums converges to


L (finite), then we say that the series
converges to L, or has sum L. If (sn)
diverges, then we say that an diverges.
P
If a series an converges to L, we write
P


X
an = L.
n=1
In other words, convergence of
a1 + a2 + a3 + . . . is defined as convergence
of the sequence (sn) of partial sums.

Conversely, given any sequence (sn) of


numbers, we can regard it as the sequence
of partial sums of a series defined as follows:
a1 = s1
a2 = s2 − s1
...
...
...
an = sn − sn−1.
With (an) defined as above, we have
sn = a1 + a2 + . . . + an for n ≥ 1.
A few more words about the notation

X ∞
X
ar , ar .
r=1

This notation is used in two distinct senses,


which it is important to distinguish.
(a) We use it to “exhibit” or “present” a
series for consideration, as in:

“Consider the series


P 1 .” or “ .... P 1 .”
r2 n

In this usage, there is not yet any


assumption that the series in question
converges.
(b) If the series converges, we use to
denote the number which is the “sum” of
the series, i.e. the limit of partial sums.
Geometric series

Theorem Let a, b ∈ R. Then

1.
P a if |b| < 1.
abn−1 converges to 1−b

2. abn−1 diverges if |b| ≥ 1.


P
Let us prove these. For simplicity, assume
that a = 1.

First prove that


P 1 if
bn−1 converges to 1−b
|b| < 1.
Here we have

a1 = 1
a2 = b
a3 = b2
...
ak = bk−1
...
Then, since b 6= 1,

sn = a1 + . . . + an
= 1 + b + . . . + bn−1
1 − bn
= .
1−b
(CHECK!)
When |b| < 1, bn → 0 as n → ∞, and so

1
lim s =
n→∞ n
.
1−b
Now use “algebra of limits” or “heredity
properties” to deduce that, for general
value of a, for |b| < 1,
a
lim s =
n→∞ n
.
1−b
On the other hand, if |b| > 1, then |bn|
diverges as n → ∞, and so sn also diverges.

If b = 1, then sn = 1 + 1 + . . . + 1 = n, so sn
diverges.

If b = −1, then s2n−1 = 1 and s2n = 0 so sn


diverges.
A simple necessary condition

Theorem If an converges, then


P

an → 0 as n → ∞.
Proof: If ar converges then there exists s
P

such that limn→∞ sn = s.

Then also limn→∞ sn−1 = s.

Then, as n → ∞,

an = sn − sn−1 → s − s = 0.
Application.

Another proof that br does not converge


P

when |b| ≥ 1.

If it did, then we would have to have br → 0


as r → ∞. Hence |b|r → 0 as r → ∞. But
|b|r ≥ 1 for all r.
Converse is false: an tending to 0 does not
necessarily mean that the series an
P

converges.

For example,
P 1 diverges, though
n
1 → 0 as n → ∞. (A proof will be
an = n
given shortly.)
Finding sufficient conditions for a series to
converge is the main aim in what follows,
and it’s not easy. Life would be simple if it
were the case that an converges if and
P

only if an → 0, but it isn’t so.


To Repeat: We need to have an → 0 if the
series an is to converge.
P

BUT just because an tends to 0, it does


not mean the series an converges!!!
P
Is that clear??!!

Offenders will be severely dealt with.


X 1
Series
ns

Theorem The ‘harmonic series’ 1/n


P

diverges.
There’s a proof in the notes. Here’s an
alternative one.

To prove this, we show that sn = r=1 1r is


Pn

not bounded above.

Let us look at s2m , i.e. the sum of the first


2m terms.
1 1 1
1 + +( + )
2 3 4
1 1 1 1
+ ( + + + ) + ... +
5 6 7 8
1 1 1
+ ( m−1 + m−1 + . . . + m)
2 +1 2 +2 2
1 1 1
> 1+ +( + )
2 4 4
1 1 1 1
+ ( + + + ) + ... +
8 8 8 8
1 1 1
+ ( m + m + . . . + m)
2 2 2
1 1 1 1
= 1 + + + + ... +
2 2 2 2
m
= 1+ .
2

So s2m is unbounded (as m varies).


1 , and so s is
But sn+1 = sn + n+1 n
increasing in n.

So sn is unbounded.
The following generalisation of the above
result is extremely useful.

1

X
Theorem The series diverges if s 1
ns
and converges if s > 1.
How do we prove it?

First, let us prove that


P 1 is divergent for
ns
s ≤ 1.

The case s = 1 already done!


For s < 1, note that, for each n,
1 1
≥ .
ns n
Then
n 1
≥ tn,
X
sn = s
r=1 r
where
n
X 1
tn = .
r=1 r
But we have already shown that tn is
unbounded. Hence sn is unbounded also,
and so must be divergent.

Soon we shall see a generalisation of this


argument, the Comparison Test.
What about s > 1? Consider the sum of the
first 2m − 1 terms.
1 1
1 + ( s + s)
2 3
1 1 1 1
+ ( s + s + s + s) + . . . +
4 5 6 7
1 1 1
+ ( (m−1)s + m−1 s
+ ... + m s
)
2 (2 + 1) (2 − 1)
1 1
≤ 1 + (2 · s ) + (4 · s ) + . . . +
2 4
+ (2m−1 · 2−(m−1)s).
Thus we have

s2m−1 ≤ 1 + 2−(s−1) + 2−2(s−1)


+ . . . + 2−(m−1)(s−1)
= 1 + b + b2 + . . . + b(m−1),

where b = 2−(s−1) satisfies |b| < 1.


Since s2m−1 is increasing in m, we have
s2m−1 ≤ (1 − b)−1 < ∞ for all m.

1
But sn+1 = sn + (n+1) s > sn for each n.

Thus sn is increasing in n, and so the above


implies that sn ≤ (1 − b)−1 for all n.
Since the sequence (sn) is monotone and
bounded, it must converge.

In other words, we have proven that


P 1
ns
converges for s > 1, as required.
There are also exist some ‘Algebra of
Limits’ results which can be proved directly
from the same results for sequences:
Theorem Suppose an and bn converge,
P P

P∞ P∞
and that n=1 an = L and n=1 bn = M .
Then, for any real number c, the series
(an + bn) and can converge, and
P P

P∞ P∞
n=1(an + bn) = L + M and n=1 can = cL.
How do we prove these?

Pn Pn
Let sn = r=1 ar and let tn = r=1 br .

By assumption, limn→∞ sn = L and


limn→∞ tn = M .
By ‘Algebra of limits’ results for sequences,

lim
n→∞
(s n + tn ) = L + M
lim (csn) = cL.
n→∞
But . . . note that the same does not hold
for products. For example, if
n √
an = (−1) / n, then, as we will shortly see,
an converges. However, (an × an)
P P

diverges. This latter series is simply


P 1 , the
n
harmonic series.
This is not all that surprising: we cannot
express the partial sums of the product
series in terms of the partial sums of the
original series in a simple way.
Let cn = an · bn.

Pn
Let sn = r=1 ar .

Pn
Let tn = r=1 br .

Pn
Let un = r=1 cr .
Then (CHECK!)
n
X
un = ar br
r=1
n
(sr − sr−1)br
X
=
r=1
n
ar (tr − tr−1)
X
=
r=1
n
(sr − sr−1)(tr − tr−1)
X
=
r=1
= ?
Example

P 1 converges.
r(r+1)

Proof:

1
Trick: r(r+1) = 1r − r+1
1 . Then, as n → ∞,
n 1 1 1 1 1 1
= 1− + − + −
X

r=1 r(r + 1) 2 2 3 3 4
1 1
+ ... + −
n n+1
1
= 1− → 1.
n+1

This is an example of a ‘telescoping’ series.


Example A more sophisticated telescoping
series,
P r−2 .
r(r+1)(r+2)

We have
n
X r−2
sn = .
r=1 r(r + 1)(r + 2)

Now note that


r−2 3 2 1
= − − .
r(r + 1)(r + 2) r+1 r+2 r
Now
n 3 2 1 3 n 1 3
− − ) =
X X
( +3 +
r=1 r + 1 r+2 r 2 r=3 r n+1
n 1 2 2
− 2 − −
X

r=3 r n+1 n+2


1 n 1
− 1− −
X

2 r=3 r
3 3 2
= + −
2 n+1 n+1
2 1
− −1−
n+2 2
3 2 2
= − + .
n+1 n+1 n+2
Thus
3 2 2
sn = − + →0
n+1 n+1 n+2
as n → ∞.

In other words,

X r−2
= 0.
r=1 r(r + 1)(r + 2)
Note that, whether or not a series converges
does not depend where we start summing it.

However, the value of the sum does. So, for


instance,

X r−2 ∞
X r−2
=
r=2 r(r + 1)(r + 2) r=1 r(r + 1)(r + 2)
−1

1·2·3
1 1
= 0+ = .
6 6
Useful Tests for Nonnegative Series

A series is nonnegative if all its terms are


nonnegative, that is ar ≥ 0 for all r.

Later we look at series which have some


negative terms, but it’s easiest at the
moment to stick to nonnegative series.
The reason why this is ‘easy’ is that the
sequence (sn) of partial sums is increasing.

The aim now is to develop a range of tests


for convergence.
Proposition. The sequence (sn) of partial
sums converges if and only if it is bounded
above. If it is not bounded above, then
sn → ∞ as n → ∞.
Proof. Since ar ≥ 0 for all r, (sn) is
increasing. If it is bounded, then
convergence follows from the fact that a
bounded monotone sequence converges.
On the other hand, if (sn) does converge,
then it must be bounded, as any convergent
sequence is bounded. If (sn) is unbounded,
then it follows from the fact that it is
increasing that it must diverge to infinity.

So ar (ar ≥ 0) either converges or diverges


P

to infinity.
Example.
P∞ 1 converges.
r=0 r!

1 + ... +
Proof: Let sn = 1 + 1 + 2! 1 .
(n−1)!

By the above, it is enough to show that


(sn) is bounded above.
But
1 1
sn ≤ 1 + 1 + +
2 2·2
1 1
+ + . . . + n−2
2·2·2 2
≤ 1 + 2 ≤ 3.

(Think about 1 + 1
2 + 1 + . . ..)
4

We call the limit ‘e’.


We defined a series ar with r-th term ar .
P

We said that such a series is convergent if


the sequence sn of its partial sums
converges to a finite limit L, i.e.
Pn
limn→∞ sn = L, where sn = r=1 ar .

If ar converges to L, we write ar = L
P P
We have seen examples of convergent and
divergent series, and proved various results
about divergent and convergent series.

We then said we would now for some time


restrict attention to series with nonnegative
terms, i.e. ar ≥ 0 for r = 1, 2, . . ..

Our aim is to develop some tests for


convergence of such series.
Comparison Test

Theorem [Comparison Test] Let (an), (bn)


be nonnegative sequences such that an ≤ bn
for all n ≥ 1. Then

1. If bn converges, then an does also, and


P P

P∞ P∞
n=1 an ≤ n=1 bn.

2. If an diverges, then bn diverges.


P P
Let us prove (1). Let

sn = a1 + a2 + . . . + an
tn = b1 + b2 + . . . + bn.

Then, by assumption, 0 ≤ sn ≤ tn and both


(sn) and (tn) are increasing.
Now, if (tn) converges, then it is bounded
above, and so (sn) is bounded above.

Since (sn) is increasing, it must converge.

Since sn ≤ tn for all n, it is clear that


limn→∞ sn ≤ limn→∞ tn.
To prove (2), assume that ar diverges.
P

Then (sn) is unbounded above (if it were


bounded then it would converge, as it is
increasing).

This implies that (tn) is unbounded, since


sn ≤ tn, and so (tn) cannot be convergent
(it is also increasing).

It follows that br diverges.


P
When using the Comparison Test, it’s
important to use it in the right direction.

If you want to use CT to show an


P

converges, you need to find bn that you


P

know converges and which satisfies


0 ≤ an ≤ bn for all n.

If you wanted to use it to show cn


P

diverges, you need a divergent series dn


P

with cn ≥ dn ≥ 0.
Application of Comparison Test:

Using the divergence of the harmonic series


P 1 , prove that P 1 diverges if s < 1.
n ns

1 ≤ 1 for each n,
Proof: note that 0 ≤ n ns
since ns ≤ n. Now apply Statement 2 in the
Comparison Test. (Earlier, we proved this
result directly, without CT.)
The Comparison Test can be weakened
slightly as follows. (Here, what we’ve done
is replace ‘for all n’ with ‘for all sufficiently
large n’.)
Theorem Let (an), (bn) be nonnegative
sequences such that there is some constant
C > 0 and some non-negative integer N
such that an ≤ Cbn for all n ≥ N . Then

1. If bn converges, then an does also.


P P

2. If an diverges, then bn diverges.


P P
Proof: The trick is to remember that the
convergence or divergence of a series does
not depend on the values of finitely many of
its terms.

(Although, if it does converge, then the


value of its sum will be influenced by the
value of each and every individual term.)
With this in mind, define ãn = aN +n−1, and
b̃n = bN +n−1 for n = 1, 2, . . ..

Then ã1 = aN , ã2 = aN +1, ã3 = aN +2, and


so on.

And b̃1 = bN , b̃2 = bN +1, b̃3 = bN +2, and so


on.
We have assumed that an ≤ Cbn for all
n ≥ N . Therefore, by definition above,
0 ≤ ãn ≤ C b̃n for n = 1, 2, . . ..

First let us prove Statement 1: if bn


P

converges, then an converges. We shall do


P

this by deducing the convergence of ãn


P

from the convergence of (C b̃n).


P
To do so, we will use Statement 1 in the
(strong) Comparison Test.

Then we need to argue that (C b̃n)


P

converges as long as bn does. And then


P

that an converges as long as ãn does.


P P
Why does (C b̃n) converge if bn
P P

converges?

First of all, since 0 < C < ∞, (C b̃n)


P

converges if and only if b̃n converges.


P

CHECK! (This follows easily from the


definition of convergence of series, together
with algebra of limits results for sequences.)
But now b̃n converges if and only if bn
P P

converges.

To see this, note that for n ≥ 1,


n X −1
n+N −1
NX −1
NX
br − br = tn+N −1−
X
t̃n = b̃r = br ,
r=1 r=1 r=1 r=1
Pn
where tn = r=1 br .

Then t̃n converges if and only if tn


converges. (Though the respective limits
will not in general be equal.)
In Statement 1, we assume that bn
P

converges, and so it follows from the above


that b̃n converges, and hence that (C b̃n)
P P

converges.

By Statement 1 in the strong form of the


Comparison Test, ãn converges.
P
But now, for each n,
n X −1
n+N −1
NX −1
NX
ar − ar = sn+N −1−
X
s̃n = ãr = ar ,
r=1 r=1 r=1 r=1
and so the convergence of ãn implies the
P

convergence of an .
P

This completes the proof of Statement 1 in


the weak Comparison Test. Note that the
weak test does not state that
P∞ P∞
n=1 an ≤ n=1 bn.
The proof of Statement 2 is very similar,
using Statement 2 in the strong
Comparison Test.

Once again, we use the series ãn and b̃n,


P P

which satisfy ãn ≤ C b̃n for all n = 1, 2, . . ..


Since an diverges, also ãn diverges.
P P

(CHECK!)

By Statement 2 in the strong Comparison


Test, (C b̃n) diverges.
P

But this then implies that b̃n diverges.


P

(CHECK!)

The last line implies that bn diverges.


P

(CHECK!)
X n2 + 1
Example Consider . The nth
n5 + n + 1
term here behaves like 1/n3, because the
dominant term on the numerator is n2 and
the dominant term in the denominator is
n5. But this needs to be made precise.
We can formally compare the series with
1/n3 by noting that
P

n2 + 1 n2 + n2 2
5
≤ 5
= 3.
n +n+1 n n
The series 2/n3 converges because 1/n3
P P

does (this being a standard result from


above, using ‘Algebra of Limits’ results for
sequences). Hence, by the CT, the given
series converges also.
The following, more sophisticated, version
of the Comparison Test, is sometimes more
useful. We could call it the ‘Limiting’
Comparison Test, but we’ll just call it the
Comparison Test (since it subsumes the
previous versions).
Theorem [Comparison Test] Suppose
(an), (bn) are positive and that an/bn → L,
where L 6= 0 (and L is finite) as n → ∞.
Then an , bn either both converge or
P P

both diverge: that is, they have the same


behaviour, with respect to convergence.
Proof: Suppose an ≥ 0, bn ≥ 0 for all n, and
an/bn → L as n → ∞ where 0 < L < ∞.

By definition of limit, for every  > 0, there


exists an integer N = N () such that, for all
n ≥ N,
an
| − L| < .
bn

Apply this with  = L/2.


Thus there exists an integer N such that,
for all n ≥ N ,
an
| − L| < L/2.
bn
That is, for n ≥ N , we have
an L 3L
< L+ =
bn 2 2
an L L
> L− = .
bn 2 2
It follows that, for n ≥ N ,
3L
an ≤ bn (1)
2
2
bn ≤ an. (2)
L

Suppose bn converges. Then by (1) and


P

the weak form of the Comparison Test


(with C = 3L/2, 0 < C < ∞), an converges
P

also.
Suppose an converges. Then by (2) and
P

the weak form of the Comparison Test


(with C = 2/L, 0 < C < ∞), bn converges
P

also.

This completes the proof.


Xn2 + 1
Example Consider again .
n5 + n + 1
Using the limiting form of the Comparison
Test, to compare the series with 1/n3, we
P

simply observe that, since


(n2 + 1)/(n5 + n + 1) n5 + n3
3
= 5
1/n n +n+1
1 + n−2
=
1 + n−4 + n−5
→ 1 6= 0,

and since 1/n3 converges, then the given


P

series converges too.


(n−1) 3
Example Consider
P
√ .
8
n +n+2

We have
(n − 1)3 n3(1 − 1/n)3
r = r
n8 + n + 2 n8(1 + n−7 + 2n−8)
n3(1 − 1/n)3
= r
n4 1 + n−7 + 2n−8
(1 − 1/n)3
= r
n 1 + n−7 + 2n−8
Thus we have
(1 − 1/n)3 1
an = r ≥ ,
n 1 + n−7 + 2n−8 16n

for all n ≥ 2.

To see this, note that (1 − 1/n)3 ≥ 1/8 for


all n ≥ 2.

r √
Also, 1 + n−7 + 2n−8 ≤ 4 = 2.
Since the harmonic series
P 1 diverges, so
n
(n−1) 3
does the series
P
√ .
8
n +n+2
Why do we bother stating all these different
forms of Comparison Test though??

Well, because life will not always be so nice


as to have an ≤ bn for all n, as in the strong
version of the CT.

Also, there will be cases when the ratio


an/bn does not converge, as in the limiting
version of the CT, but we still may be able
to use one of the other versions.
Ratio Test

Theorem [Ratio Test] Let an be a


P

nonnegative series such that


an+1
L = n→∞
lim (L = ∞ allowed).
an
Then

1. L < 1 ⇒ an converges.
P

2. L > 1 ⇒ an diverges (This includes


P

L = ∞).
Note that the Ratio Test says nothing if
L = 1: in this case, the test is useless.

Consider the series 1/n and 1/n2. In


P P

both cases, an+1/an → 1, yet the first series


is divergent and the second convergent.
Proof Let us prove (1). So suppose that
L < 1. Evidently, we may choose an M such
that L < M < 1.

Take  in the definition of limit to be


M − L > 0.
Thus there exists N such that for n ≥ N
an+1
| − L| < M − L.
an

Hence there exists N such that


an+1
n≥N ⇒ < L + (M − L) = M.
an
In particular, aN +1 < M aN .
From this, by induction,

aN +n < M naN .

Now since the geometric series M n aN


P

converges (since 0 < M < 1), we have by


the Comparison Test that aN +n
P

converges, and hence an converges.


P
Let us now prove (2). Assume then that
the limit L > 1.

As in part (1), there exists a positive


integer N such that an+1/an > 1 for n ≥ N .
In particular, aN > 0; otherwise, this is
meaningless.

By easy induction, an > aN for all n > N , so


an 6→ 0. So an diverges.
P
X n7
Example Consider . Letting
6n
an = n7/6n, we have

an+1 (n + 1)7/6n+1
=
an n7/6n
1 (n + 1)7
=
6  n7 
1 1 7 1
= 1+ → .
6 n 6

This limit is less than 1, so the series


converges.
Ratio test for br , where b > 0.
P

We have
ar+1
= b → b, as r → ∞.
ar

If 0 < b < 1, then br converges.


P

If b > 1, then br diveregs.


P
Warning

BEWARE: an+1/an may not converge at


all, in which case the Ratio Test does not
apply!!
Root Test

Theorem [Root Test] Let an be a


P

nonnegative series, and suppose that


1/n
an → L as n → ∞ (where we allow
L = ∞). Then,

1. L < 1 ⇒ an converges.
P

2. L > 1 ⇒ an diverges (This includes the


P

case L = ∞).
X n7
Example Consider again . Here,
6n
1/n
7 n7/n (n1/n)7

n
a1/n
n =  
= = .
6n
 
6 6
1/n 1/n
Now, n → 1, so an → 1/6 as n → ∞. By
the Root Test, the series converges.

Again, note that the Root Test says


nothing about the case L = 1.
1/n
Proof: Suppose that an → L, where
L < 1.

Then, given L < M < 1, by the definition of


limit with  = M − l > 0, there exists an
1/n
integer N such that an ≤ M for all n ≥ N .

So an ≤ M n for all n ≥ N .

Hence an converges by Comparison Test,


P

since M n converges.
P
1/n
Now suppose an → L, where L > 1. Then
there exists N such that for all n ≥ N ,
1/n
an > 1. So for n ≥ N , an > 1, and so
an 6→ 0. Hence no convergence.
Integral Test

The following test draws on the


interpretation of an integral as a Riemann
sum (see your calculus courses).
Theorem [Integral Test] Let g be a
positive, decreasing, integrable (for
example, continuous) function on [1, ∞).
Rn
Let G(n) = 1 g(x) dx. Then the series
g(n) converges if and only if the sequence
P

(G(n)) converges. In other words, g(n)


P

converges if and only if the improper


R∞
integral 1 g(x) dx exists.
Take a positive integer r. Since g is
decreasing, for any t ∈ [r, r + 1],

g(r) ≥ g(t) ≥ g(r + 1).

The function g is integrable so, integrating


the above over [r, r + 1], we get
Z
r+1 Z
r+1 Z
r+1
r
g(r)dt ≥ r
g(t)dt ≥ r g(r + 1)dt
or
Z
r+1
g(r) ≥ r
g(t)dt ≥ g(r + 1).
Let us sum the above inequalities from
r = 1 to r = n to obtain
n Z
n+1 n
g(r) ≥ ≥
X X
1
g(t)dt g(r + 1)
r=1 r=1
n n+1
g(r) ≥ G(n + 1) ≥
X X
g(r)
r=1 r=2
Now the sequence G(n) is increasing, since
Rn
G(n) = 1 g(t)dt and g(t) ≥ 0.

As we have seen before, limn→∞ G(n) exists


iff (G(n)) is bounded above.

Suppose that limn→∞ G(n) exists, so G(n) is


Pn+1
bounded above. Then r=2 g(r) ≤ G(n + 1)
is bounded above, and so must converge,
since it is increasing.
Pn+1
Since r=2 g(r) converges, so does
Pn
r=1 g(r), that is g(r) is convergent.
P

Conversely, if g(r) converges, then


P

Pn
r=1 g(r) is bounded above, so
Pn
G(n + 1) ≤ r=1 g(r) is bounded above, and
so limn→∞ G(n) exists.
The following variant is useful:

Theorem [Integral Test] Let g be a


positive, decreasing, integrable (for
example, continuous) function on [a, ∞),
Rn
and suppose a ≥ 1. Let G(n) = a g(x) dx.
Then the series g(n) converges if and only
P

if the sequence (G(n)) converges. In other


words, g(n) converges if and only if the
P

R∞
improper integral a g(x) dx exists.
Proof: Exercise for you!
Example Consider 1/(n log n). We know
P

that 1/n diverges and that 1/n2


P P

converges. This series is ’between’ these


two.
To see whether it converges, we can use the
integral test. Let g(n) = 1/(n log n). Then
Z
n Z
n 1 Z
log n 1
2
g(x) dx = 2 x log x
dx = log 2 u
du,

where we have made the substitution


u = log x. So

G(n) = [log u]log n


log 2 = log log n − log log 2.

Since G(n) → ∞ as n → ∞, the series is


divergent.
Alternating series

A series is alternating if its terms are


alternately positive and negative. Such a
series takes the form ± (−1)n+1cn, where
P

cn ≥ 0.
Theorem [Leibniz Alternating Series
Test/LAST] Suppose that
an = (−1)n+1cn is an alternating series,
P P

where cn ≥ 0. Then, if (cn) is a decreasing


sequence and limn→∞ cn = 0, the series an
P

converges.
Corollary For any positive number s,
X (−1)
n+1
converges.
ns

Proof: Note that cn = 1/ns is decreasing


and limn→∞ 1/ns = 0. Now apply LAST.
Proof of LAST. Consider

s2n = (c1 − c2) + (c3 − c4) + . . . + (c2n−1 − c2n).

The bracketing shows that (s2n) is an


increasing subsequence of (sn), since each
of the bracketed terms is nonnegative.
Also, we can write

s2n = c1 − (c2 − c3) − (c4 − c5) − . . .


− (c2n−2 − c2n−1) − c2n
≤ c1 .

So (s2n) is increasing and bounded above,


so it converges to a limit, s, say.
But also we have s2n+1 = s2n + c2n+1.

By assumption, c2n+1 → 0.

Hence, by ‘Algebra of Limits’, s2n+1 → s as


n → ∞. The result follows.
Note: LAST says that if the sequence (cn)
is decreasing and tends to 0, then the series
(−1)n+1cn converges.
P

It says nothing at all if one of these two


conditions fails to hold.

This does not mean that these two


conditions are necessary for convergence of
alternating series: it just means the Leibniz
test doesn’t work in those situations.
Absolute convergence

Definition Let an be a series (in which


P

some of the terms may be negative). If


|an| converges, we say that an converges
P P

absolutely. If an converges but |an|


P P

diverges, then an is said to converge


P

conditionally.
Note that if an is a convergent series with
P

nonnegative terms, then it is absolutely


convergent.
Theorem If a series an is absolutely
P

convergent, then it is convergent.


Proof. Define

a+
n = max{an, 0}
a−
n = max{−an, 0}.

Clearly, the sequences (a+ ) and (a −) are


n n
both non-negative. Further,

an = a+ − a −
n n
|an| = a+ + a −.
n n
To see this, note that, if an ≥ 0, then
a−
n = 0, and a +=a .
n n

If an < 0, then a+ = 0 and a − = −a .


n n n

Also note that a+ ≤ |a | and a − ≤ |a |.


n n n n

Since |an| is convergent, both a+


n and
P P

a−
n are convergent by the Comparison
P

Test.
Then, as an = a+ − a −, P
an is convergent
n n
by ‘Algebra of limits’ for series.
We have proved that the geometric series
ar n converges absolutely if |r| < 1.
P

It does not converge either absolutely or


conditionally when |r| ≥ 1.
X (−1)n
The fact (from Leibniz test) that
n
X1
converges and the earlier fact that
n
n
X (−1)
diverges, show that the series
n
converges conditionally.
Tests for absolute convergence

The Comparison, Ratio, and Root Tests


can be generalised.
Theorem [Comparison Test] Let (an), (bn)
be sequences such that |an| ≤ |bn| for all n.
Then

1. If bn converges absolutely, then an


P P

does also and |an| ≤ |bn|.


P P

2. If |an| diverges, then |bn| diverges.


P P
Proof. Apply the Comparison Test to the
non-negative series |an| and |bn|.
P P
Theorem [Ratio Test] Let an be a series
P

such that
|an+1|
L = n→∞
lim (L = ∞ allowed).
|an|
Then

1. L < 1 ⇒ an converges absolutely


P

2. L > 1 ⇒ an diverges.
P
Proof. To prove the first part, apply the
Ratio Test to the non-negative series |an|.
P

To prove the second part, note the proof of


the Ratio Test for nonnegative series shows
that, if L > 1, then |an| 6→ 0. It follows that
an 6→ 0, so the necessary condition of
convergence of an fails, and so an
P P

diverges.
Theorem [Root Test] Let an be a series,
P

and suppose that |an|1/n → L as n → ∞


(where we allow L = ∞). Then,

1. L < 1 ⇒ an converges absolutely.


P

2. L > 1 ⇒ an diverges (This includes the


P

case L = ∞).
Proof. To prove the first part, apply the
Root Test to the non-negative series |an|.
P

To prove the second part, note the proof of


the Root Test for nonnegative series shows
that, if L > 1, then |an| 6→ 0. It follows that
an 6→ 0, so the necessary condition of
convergence of an fails, and so an
P P

diverges.
Power Series

For our purposes, a power series is a series


of the form anxn, where x is a real
P

variable. Perhaps the most important


example of a power series is xn/n!, used to
P

define the exponential function.


It turns out that, for any real number x,
this series converges, and we may define the
exponential function by

X x
n
exp(x) = .
n=0 n!

X x
n
Consider the exponential series . It’s
n=0 n!
easy to show that this converges absolutely
for all x. We simply observe that

xn+1/(n + 1)! |x|


→ 0,

=
|xn/n!| n+1
for any x, and so, by the Ratio Test,
absolute convergence follows.
Less straightforward example.

Example Let’s determine exactly those


values of x for which the series xn/n is
P

convergent.
Taking an = xn/n, the ratio |an+1|/|an| is

xn+1/(n + 1)


n
= |x| → |x|.

|xn/n| n+1
The ratio test therefore tells us that the
series converges absolutely if |x| < 1, and
that it diverges if |x| > 1.

But what if |x| = 1?


x = 1: here, ratio test is useless and we
have to be more sophisticated.

Well, |x| = 1 corresponds to two cases:


x = 1 and x = −1. Treat each separately.
When x = 1, the series is the harmonic
X1
series , which we know diverges.
n
(−1)n
When x = −1, we have the series
X
.
n
This is convergent, by the Leibniz
Alternating Series Test. (Check this!)
So we have now determined exactly the
values of x where the series converges: it
converges for −1 ≤ x < 1 and diverges for
all other values of x.
A general result about power series is as
follows.

Theorem There is R such that the series


anxn converges absolutely for all
P

x ∈ (−R, R), and diverges for all x with


|X| > R. (This includes the case R = ∞).
In the case in which R is finite, what
happens at ±R is not determined by this
theorem, and has to be considered
separately. The name radius of convergence
is given to R.

It is possible to have R = 0.
Key Lemma If a n xn
0 converges then
P

anxn is absolutely convergent for any x


P

with |x| < |x0|.


Proof. We have that anxn
0 → 0 as n → ∞.

Therefore, there exists N such that


|anxn
0 | ≤ 1 for n ≥ N .

Let |x| < |x0|, and note that

n n x n
|anx | = |anx0 | · | |
x0
x n
≤ | | for all n ≥ N.
x0
But |x/x0| < 1, so | xx |n is a convergent
P
0
geometric series.

Hence, using the Comparison Test, anxn is


P

convergent.

Вам также может понравиться